Which one of the following could be all of the solos that are traditional pieces?

Facundo on February 19, 2016

Help

I dont get this game

Replies
Create a free account to read and take part in forum discussions.

Already have an account? log in

Mehran on April 15, 2016

@TheFacu you can watch the video for the setup by tapping the play button in the top right hand corner of the screen when viewing this question inside of LSATMax.

Hope it helps! Please let us know if you have any other questions.

on February 5, 2020

Video doesn't work. Can you please help.

Amanda on October 16, 2020

This video still doesn't work!

Alex on January 13, 2021

Hi! video still doesn't work

Victoria on January 28, 2021

Hi everyone,

Thank you for letting us know. We are sorry to hear that you are experiencing technical issues.

You can direct support-related issues to our support staff by tapping "support" from the left menu or by calling 855.483.7862 ext. 2 Monday to Friday from 9:00 a.m. to 6:00 p.m. PT.

In the meantime, I'm happy to explain this game.

We know that two pianists will perform solos at a recital: W and Z.

There will be five solos, each performed immediately after the other.

Each solo will be either M or T.

Therefore, our set-up looks like this:

Pianist: _ _ _ _ _
Style: _ _ _ _ _
1 2 3 4 5

Now let's go through our conditions.

Rule 1 - the third solo is T

Rule 2 - exactly two of the T pieces are performed consecutively

This means that either solo 2 or 4 must be T.

This then limits the style of the other pieces. If solo 2 is T, then solo 1 must be M otherwise there would be three T pieces performed consecutively. If solo 4 is T, then solo 5 must be M otherwise there would be three T pieces performed consecutively.

Rule 3 - in the fourth solo, either W performs a T piece or Z performs a M piece

If W performs a T piece, then the second and fifth solo must be M.

If Z performs a M piece, then the first solo must also be M and the second solo must be T.

Rule 4 - the pianist who performs the second solo does not perform the fifth solo

Rule 5 - no T piece is performed until W performs at least one M piece

This means that the first solo cannot be T.

Pianist: _ _ _ _ _
Style: _ _ _ _ _
1 2 3 4 5

The question asks which of the solos could be T pieces.

We can immediately eliminate answer choice (A) because we know from Rule 5 that T cannot be performed first.

We can also eliminate answer choice (B) because this would mean that there would be three T pieces performed consecutively, violating Rule 2.

We can eliminate answer choice (E) because we know from Rule 1 that the third solo is a T piece.

Finally, we can eliminate answer choice (D) because this violates Rule 2. The two T pieces must be performed consecutively.

Therefore, answer choice (C) seems like it should be the correct answer but let's double check.

Pianist: W Z Z W W
Style: M M T T M
1 2 3 4 5

Notice that this meets all of our conditions:

1) The third solo is a T piece.
2) Exactly two of the T pieces are performed consecutively (pieces three and four).
3) In the fourth solo, W performs a T piece.
4) Z performs the second solo and W performs the fifth solo.
5) W performs one M piece before a T piece is performed.

Hope this helps! Please let us know if you have any further questions.